LSAT and Law School Admissions Forum

Get expert LSAT preparation and law school admissions advice from PowerScore Test Preparation.

User avatar
 Dave Killoran
PowerScore Staff
  • PowerScore Staff
  • Posts: 5853
  • Joined: Mar 25, 2011
|
#27054
Complete Question Explanation
(The complete setup for this game can be found here: lsat/viewtopic.php?t=3418)

The correct answer choice is (C)

If P is not selected to attend the retirement dinner, then N must be selected, and when N is selected then L must be selected as well. The only remaining variable choices to consider are the pairings of J or K, and M or Q. Of these four variables, only one combination—J and Q—is impossible, and so there are three possible combinations when P is not selected:
Oct 03_M12_game#2_L5_explanations_game#3_#12_diagram_1.png
Answer choice (C) is thus correct.
You do not have the required permissions to view the files attached to this post.

Get the most out of your LSAT Prep Plus subscription.

Analyze and track your performance with our Testing and Analytics Package.